How Do You Solve an Euler-Cauchy Equation Like x^2y'' + xy' - y = 1/x?

Click For Summary
To solve the Euler-Cauchy equation x^2y'' + xy' - y = 1/x, the correct approach begins with identifying coefficients a and b, where a should be +1 instead of -1. The characteristic equation m(m-1) + am + b = 0 leads to m^2 - 2m - 1 = 0. After determining the roots, variation of parameters can be used to find the particular solution. Alternatively, the method of undetermined coefficients is applicable, although it may be more challenging to determine the correct form for the particular solution. A change of variable u = ln(x) can simplify the equation to a constant coefficients form.
engineer_dave
Messages
35
Reaction score
0

Homework Statement



Find the general solution of x^2y" + xy' - y = 1/x


Homework Equations



m(m-1) +am + b = 0 to solve an Euler Cauchy equation

The Attempt at a Solution



a=1 b=-1

m(m-1) -m -1 =0

m^2 - 2m -1 = 0

I just want to know whether the first step is right. And once I find out the values of M, do I use variation of parameters to find the particular solution? Thanks
 
Physics news on Phys.org
engineer_dave said:

Homework Statement



Find the general solution of x^2y" + xy' - y = 1/x


Homework Equations



m(m-1) +am + b = 0 to solve an Euler Cauchy equation

The Attempt at a Solution



a=1 b=-1

m(m-1) -m -1 =0

m^2 - 2m -1 = 0

I just want to know whether the first step is right. And once I find out the values of M, do I use variation of parameters to find the particular solution? Thanks
One obvious error: a= +1 here , not -1. Yes, you can use "variation of parameters". Also, because the right hand side is a power of x, you could use "undertermined coefficients" although it is slightly harder to "guess" the correct form for a particular solution in such an equation as compared to a "constant coefficients" equation. Finally, the change of variable u= ln(x) will convert this equation to a "constant coefficients" equation.
 
Question: A clock's minute hand has length 4 and its hour hand has length 3. What is the distance between the tips at the moment when it is increasing most rapidly?(Putnam Exam Question) Answer: Making assumption that both the hands moves at constant angular velocities, the answer is ## \sqrt{7} .## But don't you think this assumption is somewhat doubtful and wrong?

Similar threads

  • · Replies 18 ·
Replies
18
Views
3K
  • · Replies 6 ·
Replies
6
Views
2K
  • · Replies 2 ·
Replies
2
Views
2K
  • · Replies 2 ·
Replies
2
Views
919
  • · Replies 7 ·
Replies
7
Views
2K
Replies
6
Views
3K
Replies
2
Views
1K
  • · Replies 3 ·
Replies
3
Views
2K
  • · Replies 7 ·
Replies
7
Views
1K
  • · Replies 2 ·
Replies
2
Views
2K